0

Introduction to geometric progressions - class-X

Attempted 0/87 Correct 0 Score 0

The geometric sequence is also called as

  1. geometric progression

  2. arithmetic sequence

  3. harmonic sequence

  4. geometric series


Correct Option: A
Explanation:

The sequence is also called as Progression.

So, the geometric sequence can be called as the geometric progression.

A progression of the form $a, ar, ar^2$, ..... is a

  1. geometric series

  2. harmonic series

  3. arithmetic progression

  4. geometric progression


Correct Option: D
Explanation:

A progression of the form $a, ar, ar^2$, ..... is a geometric progression.
Geometric Progression refers to a sequence in which successor term of each term is obtained by multiplying a constant term.

The geometric progression which have infinite terms is called

  1. finite geometric progression

  2. finite arithmetic progression

  3. infinite geometric progression

  4. finite harmonic progression


Correct Option: C
Explanation:

The geometric progression which have infinite terms is called infinite geometric progression.
$1 + 0.5 + 0.25 + 0.125....$ is an example of infinite geometric progression.

If $a, b, c$ are in G.P., then

  1. $a(b^{2} + a^{2}) = c(b^{2} + c^{2})$

  2. $a(a^{2} + c^{2}) = c(a^{2} + b^{2})$

  3. $a^{2}(b + c) = c^{2}(a + b)$

  4. None of these


Correct Option: B
Explanation:

$b^{2} = ac$ satisfies (ii).

The sum $1+\dfrac { 2 }{ x } +\dfrac { 4 }{ { x }^{ 2 } } +\dfrac { 8 }{ { x }^{ 3 } } +....\left( up\ to\ \infty  \right) ,x\neq 0,$ is finite if

  1. $\left| x \right| < 2$

  2. $\left| x \right| > 2$

  3. $\left| x \right| < 1$

  4. $2\left| x \right| < 1$


Correct Option: B
Explanation:
$1 + \dfrac{2}{x} + \dfrac{4}{{{x^3}}} + \dfrac{8}{{{x^3}}}.......\,upto\,\,\infty $
It is infinite $G.P$ with Common ratio $r=\dfrac{2}{x}$
It's sum is infinite if $\left| r \right| < 1$
i-e    
 $\left| {\dfrac{2}{x}} \right| < 1$
 $ \Rightarrow \dfrac{2}{{\left| x \right|}} < 1$
 $ \Rightarrow 2 < \left| x \right|$
$ \Rightarrow \left| x \right| > 2$       

The sum of the infinite series $1+\dfrac{1}{2}+\dfrac{1}{4}+\dfrac{1}{8}+......$

  1. Cannot be determined.

  2. Equals $\dfrac{15}{8}$

  3. Equals $2$

  4. Will be higher than $2$.


Correct Option: C
Explanation:

$Given\>series\>is\>an\>infinite\>GP\>with\>first\>term\>=1\>and\>common\>ratio=1/2\\\therefore\>sum=(\frac{a}{1-r})\\=(\frac{a}{1-1/2})=2$

$S = {3^{10}} + {3^9} + \frac{{{3^9}}}{4} + \frac{{{3^7}}}{2} + \frac{{{{5.3}^6}}}{{16}} + \frac{{{3^2}}}{{16}} + \frac{{{{7.3}^4}}}{{64}} + .........$ upto infinite terms, then $\left( {\frac{{25}}{{36}}} \right)S$ equal to 

  1. ${6^9}$

  2. ${3^{10}}$

  3. ${3^{11}}$

  4. ${2.3^{10}}$


Correct Option: B
Explanation:

$\begin{array}{l}S = {3^{10}} + {3^9} + \cfrac{{{3^9}}}{4} + \cfrac{{{3^7}}}{2} + \cfrac{{{{5.3}^6}}}{{16}} + \cfrac{{{3^2}}}{{16}} + \cfrac{{{{7.3}^4}}}{{64}} + .........\infty \S = \cfrac{{1 \times {3^{10}}}}{{{2^0}}} + \cfrac{{2 \times {3^9}}}{{{2^1}}} + \cfrac{{3 \times {3^8}}}{{{2^2}}} + \cfrac{{4 \times {3^7}}}{{{2^3}}} + \cfrac{{5 \times {3^6}}}{{{2^4}}} + \cfrac{{6 \times {3^5}}}{{{2^5}}} + \cfrac{{7 \times {3^4}}}{{{2^6}}} + .....\infty \\cfrac{S}{6} = \cfrac{{{3^9}}}{2} + \cfrac{{2 \times {3^8}}}{{{2^2}}} + .......\infty \S - \cfrac{S}{6} = \cfrac{{{3^{10}}}}{2^0} + \cfrac{{{3^9}}}{{{2^1}}} + \cfrac{{{3^8}}}{{{2^2}}} + ........\infty \\cfrac{{6S - S}}{6} = \cfrac{{{3^{10}}}}{{1 - \cfrac{1}{6}}} = \cfrac{{{3^{10}}}}{5}\left( 6 \right) \end{array}$

$\dfrac56S=\dfrac{3^{10}\times 6 }{5}$
$\therefore \dfrac {25}{36}S=3^{10}$

If $4,64,p$ re in GP find p

  1. 1024

  2. 2944

  3. 512

  4. 256


Correct Option: A
Explanation:

$4,64,p$ are in GP 

Condition to be in GP is 
$b^2=ac\64^2=4p\p=\dfrac{64^2}{4}=1024$

In each of the following questions, a series of number is given which follow certain rules. One of the number is missing. Choose the missing number from the alternatives given below and mark it on your answer-sheet as directed. $1, \dfrac {1}{3}, \dfrac {1}{9}, \dfrac {1}{27}, \dfrac {1}{81}, \dfrac {1}{243}, $?

  1. $\dfrac {1}{729}$

  2. $\dfrac {1}{829}$

  3. $\dfrac {1}{749}$

  4. $\dfrac {1}{769}$


Correct Option: A
Explanation:

G.P series with common ratio $\dfrac 13$

$\therefore$, next term is $\dfrac {1}{243} \times \dfrac {1}{3}=\dfrac {1}{729}$

Find the sum of an infinite G.P : $\displaystyle 1+\frac{1}{3}+\frac{1}{9}+\frac{1}{27}+.......$

  1. $\displaystyle \frac{3}{5}$

  2. $\displaystyle \frac{3}{2}$

  3. $\displaystyle \frac{49}{27}$

  4. $\displaystyle \frac{8}{5}$


Correct Option: B
Explanation:

Given series is $1+ \dfrac {1}{3}+ \dfrac {1}{9 }+ \dfrac {1}{27}+......$

$a=1, r= \dfrac {1}{3}$

$\therefore S _{\infty}=\dfrac{a}{1-r}$

$S _{\infty}=\dfrac{1}{1-\dfrac{1}{3}}$

$S _{\infty}=\dfrac{1}{\dfrac{2}{3}}$

$\therefore S _{\infty}=\dfrac{3}{2}$

Find the GP whose $5^{th}$ term is $48$ and $9^{th}$ term is$ 768$.

  1. $3,6,12,24$

  2. $2,4,8,16$

  3. $6,12,24,48$

  4. $12,24,36,48$


Correct Option: A
Explanation:

$\displaystyle { ar }^{ 4 }=48$
$\displaystyle { ar }^{ 8 }=768$
$\displaystyle \therefore \quad { r }^{ 4 }=16$
$\displaystyle \therefore \quad r=2$
$\displaystyle a.{ 2 }^{ 4 }=48$
or, $\displaystyle a=\frac { 48 }{ 16 } =3$
The GP is 3,6, 12,24,.....

The reciprocals of all the terms of a geometric progression form a ________ progression.

  1. AP

  2. HP

  3. GP

  4. AGP


Correct Option: C
Explanation:
Let  $ a $ be the first term  and $ r $ be the common ratio of the GP. 

So, the series is $ a, ar, ar^2... $

Their reciprocals are $ \dfrac {1}{a}, \dfrac {1}{ar}, \dfrac {1}{ar^2} .. $

It is also a GP, with first term $ \dfrac {1}{a} $ and common ratio $ \dfrac {1}{r} $

In a _______ each term is found by multiplying the previous term by a constant.

  1. arithmetic sequence

  2. geometric series

  3. arithmetic series

  4. harmonic progression


Correct Option: B
Explanation:

geometric series is a series for which the ratio of each two consecutive terms is a constant function of the summation index .

Or,
In a Geometric series each term is found by multiplying the previous term by a constant.
$(Ans \to B)$

A _________ is a sequence of numbers where each term in the sequence is found by multiplying the previous term with a unchanging number called the common ratio.

  1. geometric progression

  2. arithmetic series

  3. arithmetic progression

  4. harmonic progression


Correct Option: A
Explanation:

A geometric progression is a sequence of numbers where each term in the sequence is found by multiplying the previous term with a with a unchanging number called the common ratio.
Example: $2, 6, 18, 54, 108....$
This geometric sequence has a common ratio $3$.

$10,20,40,80$ is an example of

  1. fibonacci sequence

  2. harmonic sequence

  3. arithmetic sequence

  4. geometric sequence


Correct Option: D
Explanation:

$10, 20, 40, 80$ is an example of geometric sequence.
In geometric sequence, the ratio of succeeding term to the preceeding term is always equal.

Here the common ratio is $2$.

$5 + 25 + 125 +.....$ is an example of 

  1. arithmetic progression

  2. arithmetic series

  3. geometric series

  4. geometric sequence


Correct Option: C
Explanation:

In geometric series, the ratio should be equal.
Here $5 + 25 + 125 +....$ is an example of  geometric series as their common ratio is $5$.

A ______ is the sum of the numbers in a geometric progression.

  1. arithmetic progression

  2. arithmetic series

  3. geometric series

  4. geometric sequence


Correct Option: C
Explanation:

A geometric series is the sum of the numbers in a geometric progression.

Identify the geometric series.

  1. $1 + 3 + 5 + 7 +....$

  2. $2 + 12 + 72 + 432...$

  3. $2 + 3 + 4 + 5 +...$

  4. $11 + 22 + 33 + 44+...$


Correct Option: B
Explanation:

Geometric series is of the following form:

$a+ar+ar^2+ar^3 +ar^4+..........+ar^n$
Series $2+12+72+432+......$ follows the same with $a=2$ and $r=6$.
Hence, option B is correct.

The sequence $6, 12, 24, 48....$ is a

  1. geometric series

  2. arithmetic sequence

  3. geometric progression

  4. harmonic sequence


Correct Option: C
Explanation:

The sequence $6, 12, 24, 48....$ is a geometric progression as the ratio here is common.
The common ratio in the given series is $2$.

$1, 3, 9, 27, 81$ is a

  1. geometric sequence

  2. arithmetic progression

  3. harmonic sequence

  4. geometric series


Correct Option: A
Explanation:

$1, 3, 9, 27, 81$ is a geometric sequence.
A geometric progression is a sequence of numbers such that the quotient of any two successive members of the sequence is a constant called the common ratio of the sequence.

$4, \dfrac{8}{3}, \dfrac{16}{9}, \dfrac{32}{27}..$ is a

  1. arithmetic sequence

  2. geometric sequence

  3. geometric series

  4. harmonic sequence


Correct Option: B
Explanation:

lets check the ratio between the consecutive terms.
$\dfrac {\frac {8}{3}}{4}=\dfrac {8}{12}=\dfrac {2}{3}$
Again take the ratio between next consecutive terms.
$\dfrac {\frac {16}{9}}{\frac {8}{3}}=\dfrac {16\times 3}{9\times 8}=\dfrac {2}{3}$
Here the common ratio is same $\dfrac{2}{3}$ throughout.
Hence, $4, \dfrac{8}{3}, \dfrac{16}{9}, \dfrac{32}{27}..$ is a geometric sequence.

In a _______ each term is found by multiplying the previous term by a constant.

  1. geometric sequence

  2. arithmetic sequence

  3. geometric series

  4. harmonic sequence


Correct Option: A
Explanation:
Sol:
We know that if a,b,c are in G.p then $b^2=ac$
We know that a G.P
$a,ar,ar^2.ar^3-----ar^n$
${ a } _{ 1 }{ ,a } _{ 2 },{ a } _{ 3 },{ a } _{ 4 },----{ a } _{ n }$
$\dfrac { { a } _{ 2 } }{ { a } _{ 1 } } =\dfrac { { a } _{ 3 } }{ { a } _{ 2 } } =\dfrac { { a } _{ 3 } }{ { a } _{ 3 } } ----\dfrac { { a } _{ n } }{ { a } _{ n-1 } } =r$  (r=constant)
Therefore in a geometric progression each term is found multiplying the previous term by constant .

If a sequence of values follows a pattern of multiplying a fixed amount times each term to arrive at the following term, it is called a: 

  1. geometric sequence

  2. arithmetic sequence

  3. geometric series

  4. harmonic sequence


Correct Option: A
Explanation:
$3,3^2,3^3,3^4,....(r=3)$
In a sequence if a fixed amount/constant is multiplied to each term to get the successive term the sequence is called geometric sequence.
Here $3$ is the constant which gets multiplied to each term to obtain the successive term.

Identify the geometric progression.

  1. $1, 3, 5, 7, 9, ...$

  2. $2, 4, 6, 8, 10...$

  3. $5, 10, 15, 25, 35..$

  4. $1, 3, 9, 27, 81...$


Correct Option: D
Explanation:

A geometric sequence is a sequence of numbers such that the quotient of any two successive members of the sequence is a constant called the common ratio of the sequence.
So, $1, 3, 9, 27, 81...$ is a geometric progression.
Here the common ratio is $3$.

A sequence of numbers such that the quotient of any two successive members of the sequence is a constant called the common ratio of the sequence is known as:

  1. geometric series

  2. arithmetic progression

  3. harmonic sequence

  4. geometric sequence


Correct Option: D
Explanation:

A sequence of number, ${ a } _{ 1 }+{ a } _{ 2 }+......{ a } _{ n }$ quotient of any two successive number is a constant,

$\cfrac { { a } _{ 2 } }{ { a } _{ 1 } } =\cfrac { { a } _{ 3 } }{ { a } _{ 2 } } =........=\cfrac { { a } _{ n } }{ { a } _{ n-1 } } =$common ratio $(r)$
So we can write
${ a } _{ 1 }+{ a } _{ 1 }r+{ a } _{ 2 }r+{ a } _{ 3 }r.......{ a } _{ n-1 }r\ ={ a } _{ 1 }+{ a } _{ 1 }r+{ a } _{ 1 }{ r }^{ 2 }..........{ a } _{ n-2 }{ r }^{ 2 }$
and in the end in terms of ${ a } _{ 1 }$
$={ a } _{ 1 }+{ a } _{ 1 }r+{ a } _{ 1 }{ r }^{ 2 }+{ a } _{ 1 }{ r }^{ 3 }.........{ a } _{ 1 }{ r }^{ n-1 }$
We can clearly say this series is in $GP$.
Answer $(D)$

Which one of the following is not a geometric progression?

  1. $1, 2, 4, 8, 16, 32$

  2. $4, -4, 4, -4, 4$

  3. $12, 24, 36, 48$

  4. $6, 12, 24, 48$


Correct Option: C
Explanation:
For geometric progression, the ratio of the consecutive terms should be equal.
Here $12, 24, 36, 48$ is not a geometric progression. Here only the difference is common i.e. $12$.
Rest all options have same common ratio i.e., in option A, the ratio is $2$. In option B, the ratio is $-1$.
And in option D, the ratio is $2$.
Here the given sequence is an arithmetic progression.

Which one of the following is a geometric progression?

  1. $3, 5, 9, 11, 15$

  2. $4, -4, 4, -4, 4$

  3. $12, 24, 36, 48$

  4. $6, 12, 24, 36$


Correct Option: B
Explanation:

$4, -4, 4, -4, 4$ is a geometric progression.
Here the common ratio is $-1$.

Option B is correct.

Which of the following is not in the form of G.P.?

  1. $2 + 6 + 18 + 54 +...$

  2. $3 + 12 + 48 + 192 +....$

  3. $1 + 4 + 7 + 10 +....$

  4. $1 + 3 + 9 + 27 +....$


Correct Option: C
Explanation:

In option A, the common ratio is $3$.
In option B, the common ratio is $4$.
In option D, the common ratio is $3$.
$1 + 4 + 7 + 10 +...$. is not a G.P., since the sequence is in the form of A.P.

Which one of the following is a general form of geometric progression?

  1. $1, 1, 1, 1, 1$

  2. $1, 2, 3, 4, 5$

  3. $2, 4, 6, 8, 10$

  4. $-1, 2, -3, 4, -5$


Correct Option: A
Explanation:
Lets see option A:
Sequence is $1,1,1,1,1$
General form of GP is $a=1$ and $r=1$
Here ratio is constant throughout.
Thus in all options, option A is correct.
The geometric progression is $1,1,1,1,1,............$.

The number of terms in a sequence $6, 12, 24, ....1536$ represents a

  1. arithmetic progression

  2. harmonic progression

  3. geometric progression

  4. geometric series


Correct Option: C
Explanation:
Given series is $6,12,24,....1536$
Since, $\dfrac {12}{6} =2$ and $\dfrac {24}{12} =2$
i.e. the given sequence is a geometric sequence / progression. 

Find out the general form of geometric progression.

  1. $2, 4, 8, 16$

  2. $2, -2, 2, 3, 1$

  3. $0, 3, 6, 9, 12$

  4. $10, 20, 30, 40$


Correct Option: A
Explanation:

The general form of geometric progression is $2, 4, 8, 16$.

Because here common ration between the consecutive terms is same. That is illustrated below.
$\dfrac {4}{2}=2, \dfrac {8}{4}=2, \dfrac {16}{8}=2$
Here the common ratio is $2$.

For which sequence below can we use the formula for the general term of a geometric sequence?

  1. $1, 3, 5, 7, 9.....$

  2. $2, 4, 6, 8, 10.....$

  3. $4, 8, 16, 32, 64....$

  4. $1, -1, 3, -2, 4$


Correct Option: C
Explanation:

For a G.P., the ratio must be common throughout.
We use the formula for the general term of a geometric sequence for $4, 8, 16, 32, 64.... $
Here the common ratio is $2$.

An example of G.P. is

  1. $-1, \dfrac{1}{2}, \dfrac{1}{4}, \dfrac{1}{8}...$

  2. $ -1, \dfrac{3}{2}, \dfrac{1}{2}, -\dfrac{1}{2}$

  3. $1, \dfrac{1}{2}, \dfrac{1}{4}, \dfrac{1}{6}...$

  4. $1, \dfrac{1}{2}, \dfrac{1}{4}, \dfrac{1}{8}...$


Correct Option: D
Explanation:

For a G.P., the ratio must be equal.
Here only D satisfies thiss condition.
So, an example of G.P. is $1, \dfrac{1}{2}, \dfrac{1}{4}, \dfrac{1}{8}...$
Here the common ratio is $\dfrac{1}{2}$.

The common ratio is used in _____ progression.

  1. arithmetic

  2. geometric

  3. harmonic

  4. series


Correct Option: B
Explanation:

The common ratio is used in geometric progression.

For example: $2,4,8,16,....$
Here the common ratio is $2$.

Which of the following is a general form of geometric sequence?

  1. {$2, 4, 6, 8, 10$}

  2. {$-1, 2, 4, 8, -2$}

  3. {$2, -2, 2, -2, 2$}

  4. {$3, 13, 23, 33, 43$}


Correct Option: C
Explanation:

{$2, -2, 2, -2, 2$} is a general form of geometric sequence.

For a G.P, the ratio must be equal throughout.
Here the common ratio is $-1$.

The common ratio is calculated in

  1. A.P.

  2. G.P.

  3. H.P.

  4. I.P.


Correct Option: B
Explanation:

The common ratio is calculated in G.P.
For example: $2,4,8,16,....$

Here the common ratio is $2$.

The series $a, ar, ar^2, ar^3, ar^4....$ is an

  1. finite geometric progression

  2. finite harmonic progression

  3. infinite geometric progression

  4. finite arithmetic progression


Correct Option: C
Explanation:

$a, ar, ar^2, ar^3, ar^4....$ is an infinite geometric progression.

Here common ratio is $r$.
This can be found out as $\dfrac {ar}{a}=r, \dfrac {ar^2}{ar}=r$ and so on.
Thus the given series is in G.P.

The general form of GP $a, ar, ar^2, ar^3, ar^4$ is a

  1. finite geometric progression

  2. finite harmonic progression

  3. infinite geometric progression

  4. finite arithmetic progression


Correct Option: A
Explanation:

Given sequence is $a,ar,ar^2, ar^3, ar^4$.

It is the general form of a finite geometric progression as the series stops at some point of finite terms.

$1 + 0.5 + 0.25 + 0.125....$ is an example of

  1. finite geometric progression

  2. infinite geometric series

  3. finite geometric sequence

  4. infinite geometric progression


Correct Option: D
Explanation:

$1 + 0.5 + 0.25 + 0.125....$ is an example of infinite geometric progression.
Here the common ratio is $0.5$.
An infinite geometric series is the sum of an infinite geometric progression.

How will you identify the sequence is an infinite geometric progression?

  1. An geometric sequence containing finite number of terms

  2. An geometric sequence containing infinite number of terms

  3. An arithmetic sequence containing infinite number of terms

  4. An arithmetic sequence containing finite number of terms


Correct Option: B
Explanation:

An geometric sequence containing infinite number of terms. It has a common ratio which is same throughout.
Example: $1 + 0.5 + 0.25 + 0.125....$ is an infinite geometric sequence.
Here the common ratio is $0.5$.

How would you find the sequence is finite geometric sequence?

  1. An arithmetic sequence containing finite number of terms

  2. A geometric sequence containing finite number of terms

  3. An arithmetic sequence containing infinite number of terms

  4. A geometric sequence containing infinite number of terms


Correct Option: B
Explanation:

If a sequence is a finite geometric sequence, then :

It will have the finite number of terms.
it will be a geometric sequence i.e. its ratio will be constant throughout.
Option B is the correct answer.

Identify the finite geometric progression.

  1. $3, 6, 12, 24...$

  2. $81, 27, 9, 3..$

  3. $10 - 5 + 2.5 - 1.25.....$

  4. $1 + 0.5 + 0.25 + 0.125$


Correct Option: D
Explanation:

$1 + 0.5 + 0.25 + 0.125 $$ is a finite geometric progression.
Here the common ratio is $0.5$.
An finite geometric series is the sum of an finite geometric sequence.

Identify the correct sequence represents a infinite geometric sequence.

  1. $3, 6, 12, 24, 48$

  2. $1 + 2 + 4 + 8 +....$

  3. $1, -1, 1, -1, 1$

  4. $1, 3, 4, 5, 6....$


Correct Option: B
Explanation:

An infinite geometric series is the sum of an infinite geometric sequence.
So, $1 + 2 + 4 + 8 +....$ is an infinite geometric sequence.
Here the common ratio is $2$ and it is never ending.

If $\dfrac{a-b}{b-c}=\dfrac{a}{b}$, then $a, b, c $ are in

  1. GP

  2. HP

  3. AP

  4. SP


Correct Option: A
Explanation:

Given:

$\dfrac{a-b}{b-c}=\dfrac{a}{b}$
$\Rightarrow ab-b^{2}=ab-ac$
$\therefore b^{2}=ac  \rightarrow G.P.$
$(Ans \to A)$

$2+{2}^{2}+{2}^{3}+.......+{2}^{9}=$?

  1. $2044$

  2. $1022$

  3. $1056$

  4. None of these


Correct Option: B
Explanation:

This is G.P in which  $a=2,r=\cfrac{{2}^{2}}{2}=2$ and $n=9$
$\therefore$ ${S} _{n}=\cfrac{a({r}^{n}-1)}{(r-1)}=\cfrac{2\times ({2}^{9}-1)}{(2-1)}=2\times (512-1)=2\times 511=1022$.

How many terms are there in the G.P $3,6,12,24,.........,384$?

  1. $8$

  2. $9$

  3. $10$

  4. $11$

  5. $7$


Correct Option: A
Explanation:

Here $a=3$ and $r=\cfrac{6}{3}=2$. Let the number of terms be $n$$.
Then, ${t}_{n}=384$ $\Rightarrow$ $a{r}^{n-1}=384$
$\Rightarrow$ $3\times {2}^{n-1}=384$
$\Rightarrow$ ${2}^{n-1}=128={2}^{7}$
$\Rightarrow$ $n-1=7$
$\Rightarrow$ $n=8$
$\therefore$ Number of terms $=8$.

For a set of positive numbers, consider the following statements:
1. If each number is reduced by $2$, then the geometric mean of the set may not always exists.
2. If each number is increased by $2$, then the geometric mean of the set is increased by $2$.
Which of the above statements is/are correct?

  1. $1$ only

  2. $2$ only

  3. Both $1$ and $2$

  4. Neither $1$ nor $2$


Correct Option: A
Explanation:

1. Consider the two numbers $1$ and $4$, geometric mean of $1$ and $4$ is $\sqrt {1 \times 4} = 2$.
When each number is reduced by $2$, the numbers become $-1$ and $2$ whose geometric mean does not exist.
2. Now consider two numbers $2$ and $7$. Their geometric mean is $\sqrt {14}$. The new numbers are $4$ and $9$ whose geometric mean is $\sqrt {4\times 9} = 6$ which is not equal to $2\sqrt {14}$.
Thus only statement $1$ is true.

If $a, b, c$ are in G.P., then $\dfrac {a - b}{b - c}$ is equal to

  1. $\dfrac {a}{b}$

  2. $\dfrac {b}{a}$

  3. $\dfrac {a}{c}$

  4. $\dfrac {c}{b}$


Correct Option: A
Explanation:

We are given $a,b,c$ are in G.P.


Hence, ${b}^{2}=a\times c$


$\dfrac { a-b }{ b-c }=\dfrac { a-b }{ b-\dfrac { { b }^{ 2 } }{ a }  } $

$=\dfrac { a\left( a-b \right)  }{ b\left( a-b \right)  } $

$=\dfrac { a }{ b } $

Hence, option A is correct.

Say true or false.

Zero can be the common ratio of a G.P.

  1. True

  2. False


Correct Option: B
Explanation:

The common ratio between two numbers has to be non-zero for it to form a G.P.

Say true or false.
$2, 4, 8, 16, .....$ is not an $A.P.$

  1. True

  2. False


Correct Option: A
Explanation:

Difference between consecutive terms is $(4-2), (8-4), (16-8)$ and so on, i.e. 2,4,8 and so on.
Since the difference is not constant, it is not an AP.

The sum of infinity of $\frac{1}{7} + \frac{2}{7^2} + \frac{1}{7^3} + \frac{2}{7^4} + ......$ is:

  1. $\frac{1}{5}$

  2. $\frac{1}{24}$

  3. $\frac{5}{48}$

  4. $\frac{3}{16}$


Correct Option: D

The limit of the sum of an infinite number of terms in a geometric progression is $a/(1 - r)$ where a denotes the first term and $-1 <r<1$ denotes the common ratio. The limit of the sum of their squares is:

  1. $\dfrac{a^2}{(1 - r)^2}$

  2. $\dfrac{a^2}{1 + r^2}$

  3. $\dfrac{a^2}{1 - r^2}$

  4. $\dfrac{4a^2}{1 + r^2}$


Correct Option: C

If $S=1+\dfrac{1}{2}+\dfrac{1}{4}+\dfrac{1}{8}+\dfrac{1}{16}+\dfrac{1}{32}+....\infty$.
then, the sum of the given series is $2$.

  1. True

  2. False


Correct Option: A
Explanation:

We have,

$S=1+\dfrac{1}{2}+\dfrac{1}{4}+\dfrac{1}{8}+\dfrac{1}{16}+\dfrac{1}{32}+....\infty$

Then,
$a=1$, $r=\dfrac{1}{2}$

We know that
$S=\dfrac{a}{1-r}$

$S=\dfrac{1}{1-\dfrac{1}{2}}$

$S=\dfrac{1}{\dfrac{1}{2}}$

$S=2$

Hence, this is the answer.

Given a sequence of $4$ members, first three of which are in G.P. and the last three are in A.P. with common difference six. If first and last terms of this sequence are equal, then the last term is:

  1. $8$

  2. $16$

  3. $2$

  4. $4$


Correct Option: A
Explanation:
Let for terms be $a,ar,ar^{2},a$

$\because ar,ar^{2}$ and a are in A.P

$\therefore ar^{2}-ar=6$

$\Rightarrow ar(r-1)=6$

And $a-ar=2\times 6$

$\Rightarrow a(r-1)=-12$

$\Rightarrow \dfrac{ar(r-1)}{a(r-1)}=\dfrac{-6}{12}$

$\Rightarrow r=-\dfrac{1}{2}(\because r\neq 1)$

$\therefore a(1-r)=12$

$\Rightarrow a\left(1+\dfrac{1}{2}\right)=12$

$\Rightarrow \dfrac{39}{2}=12$

$\Rightarrow a=8$

$\therefore $ Last term = $8$

$n$ is an integer. The largest integer $m$, such that ${n^m} + 1$ divides $1 + n + {n^2} + .....{n^{127}},$ is

  1. $127$

  2. $63$

  3. $64$

  4. $32$


Correct Option: C
Explanation:

$1+n+{ n }^{ 2 }+...{ n }^{ 127 }=\cfrac { { n }^{ 128 }-1 }{ n-1 } \ =\cfrac { \left( { n }^{ 64 }-1 \right) \left( { n }^{ 64 }+1 \right)  }{ (n-1) } \ ({ a }^{ n }-{ b }^{ m })$ is divisible by $\quad (a-b)\bigvee  m\in { z }^{ + }$(positive Integers)

$\left( { n }^{ 64 }-1 \right) $ is divisible by $(n-1)$
$\cfrac { \left( { n }^{ 64 }-1 \right)  }{ (n-1) } $ is Integer value
$1+n+{ n }^{ 2 }+...{ n }^{ 127 }$ is divisible by $\left( { n }^{ 64 }+1 \right) $
Given $1+n+{ n }^{ 2 }+...{ n }^{ 127 }$ is divisible by $\quad { n }^{ m }+1$
Maximum possible value of m $m=64$

Tangent at a point ${P _1}$ (other than (0, 0) on the curve $y = {x^3}$ meets the curve again at ${P _2}$. The tangent at ${P _2}$ meets the curve again at ${P _3}$ and so on. Show that the abscissae of ${P _1},{P _2},..........,{P _n}$ form a G.P. Also find the ratio $\left[ {area\,\left( {\Delta {P _1}.{P _2}.{P _3}} \right)/area\,\left( {\Delta {P _2}{P _3}{P _4}} \right)} \right].$

  1. $\dfrac{1}{2}$

  2. $\dfrac{1}{4}$

  3. $\dfrac{1}{8}$

  4. $\dfrac{1}{16}$


Correct Option: D

If $a, b, c$ are in G.P., then

  1. $a^2, b^2, c^2$ are in G.P.

  2. $a^2(b+c), c^2 (a+b), b^2 (a+c)$ are in G.P.

  3. $\displaystyle \frac{a}{b+c}, \frac{b}{c+a}, \frac{c}{a+b}$ are in G.P.

  4. None of the above.


Correct Option: A
Explanation:

Given:  a, b, c are in G.P.

$\therefore b^{2}=ac$                                    ...eq ( 1 )
Squaring both sides:
$\Rightarrow b^{4}=a^{2}c^{2}$
$\Rightarrow (b^{2})^{2}=a^{2}c^{2}$
$\therefore a^{2},b^{2},c^{2}$ are in G.P.                    

Consider an infinite $G.P$. with first term $a $ and common ratio $r$, its sum is $4$ and the second term is $\dfrac {3}{4}$, then?

  1. $a=\dfrac{4}{7}, r=\dfrac{3}{7}$

  2. $a=\dfrac{3}{2}, r=\dfrac{1}{2}$

  3. $a=1, r=\dfrac{3}{4}$

  4. $a=3, r=\dfrac{1}{4}$


Correct Option: C,D
Explanation:

Given:-

${S} _{\infty} = 4$
${a} _{2} = \cfrac{3}{4}$
$\Rightarrow ar = \cfrac{3}{4}$
$\Rightarrow 4ar = 3 ..... \left( 1 \right)$
As we know that,
${S} _{\infty} = \cfrac{a}{1 - r}$
$\therefore \cfrac{a}{1 - r} = 4$
$\Rightarrow 4r = 4 - a ..... \left( 2 \right)$
From equation $\left( 1 \right) &amp; \left( 2 \right)$, we have
$a \left( 4 - a \right) = 3$
$\Rightarrow {a}^{2} - 4a + 3 = 0$
$\Rightarrow \left( a - 3 \right) \left( a - 1 \right) = 0$
$\Rightarrow a = 1$ or $a = 3$
Substituting the value of $a$ in equation $\left( 1 \right)$, we get
For $a = 3$
$\Rightarrow r = \cfrac{1}{4}$
For  $a = 1$
$\Rightarrow r = \cfrac{3}{4}$

The first term of an infinite geometric progression is x and its sum is $5$. then 

  1. $x < -10$

  2. $0 < x < 10$

  3. $-10 < x < 10$

  4. $x > 10$


Correct Option: B
Explanation:
Given:-
${S} _{\infty} = 5$
$a = x$
As we know that,
${S} _{\infty} = \cfrac{a}{1 - r}$
$\therefore \cfrac{x}{1 - r} = 5$
$\Rightarrow \cfrac{x}{5} = 1 - r$
$\Rightarrow r = 1 - \cfrac{x}{5}$
Now,
$\left| r \right| < 1$
$\left| 1 - \cfrac{x}{5} \right| < 1$
$\Rightarrow -1 < 1 - \cfrac{x}{5} < 1$
$\Rightarrow -2 < \cfrac{-x}{5} < 0$
$\Rightarrow 0 < \cfrac{x}{5} < 2$
$\Rightarrow 0 < x < 10$

The first three of four given numbers are in G.P. and last three are in A.P. whose common difference is $6$. If the first and last numbers are same, then first will be?

  1. $2$

  2. $4$

  3. $6$

  4. $8$


Correct Option: D
Explanation:

Last $3$ of the $4$ numbers are in AP.


Let they are, $a-d, a, a+d$. Also the first number is same as $4th, a+d$.

Therefore the $4$ numbers are $a+d, a-d, a, a+d$

The first $3$ of these are in G.P.


$\therefore (a-d)^2=a(a+d)$ But $d=6$

$\therefore (a-6)^2=a(a+6)$

$\therefore a^2-2a+36=0$

Solving the above quadratic equation, we get,

$a=2$

Therefore the series is:

$8,-4,2,8$


The sum of $1 + \left( {1 + a} \right)x + \left( {1 + a + {a^2}} \right){x^2} + ....\infty ,\,0 < a,\,x < 1$ equals

  1. $\dfrac{1}{{\left( {1 - x} \right)\left( {1 - a} \right)}}$

  2. $\dfrac{1}{{\left( {1 - a} \right)\left( {1 - ax} \right)}}$

  3. $\dfrac{1}{{\left( {1 - x} \right)\left( {1 - ax} \right)}}$

  4. $\dfrac{1}{{\left( {1 - x} \right)\left( {1 + a} \right)}}$


Correct Option: C
Explanation:
$1+\left( 1+a \right) x+\left( 1+a+{ a }^{ 2 } \right) { x }^{ 2 }+\dots \infty $

$ General\quad term\quad of\quad series={ T } _{ n }=\frac { \left( 1-{ a }^{ r+1 } \right)  }{ \left( 1-a \right)  } { x }^{ r }$  

$ { S } _{ n }=\sum _{ r=1 }^{ \infty  }{ \dfrac { \left( 1-{ a }^{ r+1 } \right) { x }^{ r } }{ \left( 1-a \right)  }  } $  

$ \left( 1-a \right) { S } _{ n }=\sum _{ r=1 }^{ \infty  }{ { x }^{ r } } -a\sum _{ r=1 }^{ \infty  }{ \left( ax \right) ^{ r } } $ 

$\left( 1-a \right) { S } _{ n }=\dfrac { 1 }{ 1-x } -\dfrac { a }{ 1-ax } $     ...........  $\because G.P.$ series 

$ \left( 1-a \right) { S } _{ n }=\dfrac { 1-ax-a+ax }{ \left( 1-x \right) \left( 1-ax \right)  } $  

$ \left( 1-a \right) { S } _{ n }=\dfrac { 1-a }{ \left( 1-x \right) \left( 1-ax \right)  } $  

$ { S } _{ n }=\dfrac { 1 }{ \left( 1-x \right) \left( 1-ax \right)  } $
B is correct

If roots of the equations $(b-c)x^2+(c-a)x+a-b=0$, where $b\neq c$, are equal, then a, b, c are in?

  1. G.P.

  2. H.P.

  3. A.P.

  4. A.G.P.


Correct Option: C
Explanation:
$(b-c)x^{2}+(c-a)x+a-b=0$
Root are equal, so $D=0$
$\Rightarrow (c-a)^{2}-4(b-c)(a-b)=0$
$\Rightarrow c^2+a^2-2ac-4ab+4b^2+4ac-4bc=0$
$\Rightarrow c^2+a^2+4b^2+2ac-4ab-4bc=0$
$\Rightarrow (a+c-2b)^{2}=0$
$a+c=2b$

























If the roots of ${ x }^{ 2 }-k{ x }^{ 2 }+14x-8=0$ are in geometric progression, then $k=$

  1. $-3$

  2. $7$

  3. $4$

  4. $0$


Correct Option: B
Explanation:

$\Rightarrow \frac { a }{ r } ,a,ar=8\quad \quad \Rightarrow { a }^{ 3 }=8\quad \quad \Rightarrow a=2$
$a=2$ is a root of the given equation 
$\Rightarrow 8-4k+28-8=0\quad \quad \Rightarrow k=7$

The third term of a geometric progression is $4$. The product of the first five terms is 

  1. ${4}^{3}$

  2. ${4}^{4}$

  3. ${4}^{5}$

  4. ${4}^{6}$


Correct Option: C
Explanation:
Let $a$ and $r$ the first term and common ratio, respectively.
Given:Third term$=a{r}^{2}=4$
Product of first $5$ terms is
$=a.ar.a{r}^{2}.a{r}^{3}.a{r}^{4}$
$={a}^{5}{r}^{10}$
$={\left(a{r}^{2}\right)}^{5}$
$={4}^{5}$

If $a,\ b,\ c$ are in $G.P$, then
$a(b^{2}+c^{2})=c(a^{2}+b^{2})$

  1. True

  2. False


Correct Option: A
Explanation:

Given 


$a,b,c$ are in GP

$\implies  b^2=ac\cdots(1)$

LHS

$a(b^2+c^2)$

$\implies a(ac+c^2)$

$\implies a^2c+ac^2$

$\implies c(a^2+ac)$

$\implies c(a^2+b^2)$

RHS

Hence Proved.

In a GP the sum of three numbers is $14 ,$ if $1$ is added to first two numbers and the third number is decreased by $1$, the series becomes AP, find the geometric sequence.

  1. $2,4,8$

  2. $8,4,2$

  3. $6,18,54$

  4. $8,16,32$


Correct Option: A,B
Explanation:


$\\Let\>the\>numbers\>an\>a,\>ar,\>ar^2\\where\>a=first\>term\>and\>r=common\>ratio\\\therefore\>a+ar+ar^2=14\\and\\a+1,\>ar+1,\>ar^2-1\>are\>in\>AP\\\therefore\>2(ar+1)=(a+1)+(ar^2-1)\\or\>2ar+2=a+ar^2\\or\>3ar+2=a+ar+ar^2\\or\>3ar+2=14\\\therefore\>ar=4\\or\>a=(\frac{4}{r})\\\therefore(\frac{4}{r})+(\frac{4}{r})\times\>r+(\frac{4}{r})\times\>r^2=14\\or\>(\frac{4}{r})+4+4r=14\\or\>4+4r+4r^2=14r\\or\>4r^2-10r+4=0\\or\>4r^2-8r-2r+4=0\\or\>4r(r-2)-2(r-2)=0\\or\>(4r-2)(r-2)=0\\\therefore\>r=(\frac{1}{2})\>or\>2\\ifr=(\frac{1}{2}),then\>a=(\frac{4}{(\frac{1}{2})})=8\\\therefore\>sequence\>8,4,2\>\\\>and\>if\>r=2,\>then\>a=(\frac{4}{2})=2\\\therefore\>sequence\>2,4,8$

 

Which of the following is a geometric series? 

  1. $2,4,6,8 , \dots \dots$

  2. $1 / 2,1,2,4 \dots \dots$

  3. $1 / 4,1 / 6,1 / 8,1 / 10 , \dots \ldots$

  4. $3,9,18,36 , \dots$


Correct Option: B
Explanation:

A series is said to be geometric when ratio between one term and its previous term is constant/same throughout the series.


a) $2, 4, 6, 8$


$\Rightarrow$$\dfrac{4}{2} \neq \dfrac{6}{4} $

b) $\dfrac{1}{2} , 1 , 2, 4$

$\Rightarrow$$\dfrac{1}{\dfrac{1}{2}} = \dfrac{2}{1} = \dfrac{4}{2} = 2 $

c) $\dfrac{1}{4} , \dfrac{1}{6} , \dfrac{1}{8}$

$\Rightarrow$$\dfrac{1/6}{1/4} \neq \dfrac{1/8}{1/6}$

d) $3, 9, 18 , 36$

$\Rightarrow$$\dfrac{9}{3} \neq \dfrac{18}{9}$.

Coefficient of $x^r$ in $1+(1+x)+(1+x)^2+......+ (1+x)^n$ is 

  1. $^{n+3}C _r$

  2. $^{n+1}C _{r+1}$

  3. $^nC _r$

  4. $^{(n+2)}C _r$


Correct Option: A
Explanation:

Consider given the series, $1+\left( 1+x \right)+{{\left( 1+x \right)}^{2}}+{{\left( 1+x \right)}^{3}}+...............{{\left( 1+x \right)}^{n}}$

Given series is G.P. which have first term $a=1$,comman ratio $r=1+x$ and number of term $=n$

So sum is,

${{s} _{n}}=\dfrac{a\left( {{r}^{n}}-1 \right)}{r-1}=\dfrac{[{{\left( 1+x \right)}^{n}}-1]}{1+x}={{\left( 1+x \right)}^{n-1}}-{{\left( 1+x \right)}^{-n}}$

Now rth term of ${{\left( 1+x \right)}^{n-1}}-{{\left( 1+x \right)}^{-n}}$ is $={}^{n-1}{{C} _{r}}{{x}^{r}}-{}^{-n}{{C} _{r}}.{{x}^{r}}=\left( {}^{n-1}{{C} _{r}}-{}^{-n}{{C} _{r}} \right){{x}^{r}}$


Hence, coefficient of ${{x}^{r}}=\left( {}^{n-1}{{C} _{r}}-{}^{-n}{{C} _{r}} \right)$


Hence, this is the answer.

The value of $p$ if $3,p,12$ are in GP

  1. $6$

  2. $4$

  3. $9$

  4. None.


Correct Option: A
Explanation:

Given $3,p,12$ are in GP

The condition to be in GP
$b^2=ac\\p^2=3(12)\\p^2=36\\p=6$

The common ratio of GP $4,8,16,32,.....$ is

  1. $2$

  2. $3$

  3. $4$

  4. $0$


Correct Option: A
Explanation:

Given series is $4,8,16,32,....$

Common ratio is given as $\dfrac84=2$

If $\alpha, \beta, \gamma$ are non-constant terms in G.P and equations $\alpha { x }^{ 2 }+2\beta x+\gamma =0\quad $ and ${x}^{2}+x-1=0$ has a common root then $\left( \gamma -\alpha  \right) ,\beta $ is

  1. $\alpha \beta $

  2. $\beta \gamma $

  3. $\gamma \alpha $

  4. $0$


Correct Option: C
Explanation:

Let the common ratio of G.P is $r$ Therefore $\quad \beta =\alpha t,\alpha { t }^{ 2 }$
Equation $\alpha { x }^{ 2 }+2\alpha rx+\alpha { t }=0\quad 
\Rightarrow { x }^{ 2 }+2rx+{ t }^{ 2 }=0....(i)$
Given equation (i) and ${ x }^{ 2 }+x-1=0....(ii)$ has a common root
$(i)-(ii)\Rightarrow (2e-1)x+({ r }^{ 2 }+1)=0\Rightarrow x=\cfrac { -\left( { r }^{ 2 }+1 \right)  }{ 2r-1 } ....(iii)\quad $
Putting (iii) in equation (ii) $\Rightarrow { \left( { r }^{ 2 }+1 \right)  }^{ 2 }-\left( { r }^{ 2 }+1 \right) (2r-1)-{ \left( { 2r }^{ 2 }-1 \right)  }^{ 2 }=0\Rightarrow { r }^{ 4 }-2{ r }^{ 3 }-{ r }^{ 2 }+2r+1=0....(iv)$
dividing equation (iv) by ${r}^{2}$ $\Rightarrow { \left( r-\cfrac { 1 }{ r }  \right)  }^{ 2 }-2{ \left( r-\cfrac { 1 }{ r }  \right)  }+1=0\Rightarrow { \left( r-\cfrac { 1 }{ r } -1 \right)  }^{ 2 }=0\Rightarrow \cfrac { r-1 }{ r } =1....(v)\quad $
$\left( \gamma -\alpha  \right) \beta =\left( \alpha { r }^{ 2 }-\alpha  \right) \times \alpha r={ \alpha  }^{ 2 }\left( { \alpha  }^{ 2 }-1 \right) r={ \alpha  }^{ 2 }(r-1)={ \alpha  }^{ 2 }{ r }^{ 2 }$
(using $(v)=\alpha \times \alpha { t }^{ 2 }\quad $

Write down the first five terms of the geometric progression which has first term 1 and common ratio 4.

  1. 1, 4, 16, 64, 244

  2. 1, 4, 24, 64, 256

  3. 1, 4, 16, 32, 256

  4. 1, 4, 16, 64, 256


Correct Option: D
Explanation:

Let a and d be the first term and common ratio of the GP respectively.
Given a=1 and d=4.
Now, $a _n=ar^{n-1}$
$\therefore a _1=a=1$
$a _2=ar=1\times4=4$
$a _3=ar^2=1\times(4)^2=16$
$a _4=ar^3=1\times(4)^3=64$
$a _5=ar^4=1\times(4)^4=256$





$\displaystyle \frac{1}{c},(\frac{1}{ca})^{\dfrac{1}{2}},\frac{1}{a}$ is in

  1. AP

  2. GP

  3. HP

  4. NONE


Correct Option: B
Explanation:

Given series


$\dfrac{1}{c},\left(\dfrac{1}{ca}\right)^{\dfrac{1}{2}},\dfrac{1}{a}$

Lets consider a G.P of elements $A,B,C$

 $\therefore$ Geo.mean $\Rightarrow B^2=AC$

Comparing it with given series.

$A=\dfrac{1}{c}B=\left(\dfrac{1}{ca}\right)^{\dfrac{1}{2}},C=\dfrac{1}{a}$

$\therefore B^2=\left(\dfrac{1}{ca}\right)^{\dfrac{1}{2}\times 2}$

            $=\dfrac{1}{ca}$........(1)

$AC=\dfrac{1}{c}\times \dfrac{1}{a}=\dfrac{1}{ca}$..............(ii)

$\therefore (i)=(ii)$

$\therefore B^2=AC$ So given series is in G.P 

Determine the relations among x, y and z if $y^{2}=xz$

  1. A.P

  2. G.P

  3. A.G.P

  4. none of these


Correct Option: B
Explanation:

It is fundamental concept that if $y^2 = xz$, then $x,y,z\in$  G.P

Find the sum the infinite G.P.: $\displaystyle {\frac{2}{3}\, -\, \frac{4}{9}\, +\, \frac{8}{27}\, -\, \frac{16}{21}\, +\, ........}$ 

  1. $\displaystyle \frac{2}{5}$

  2. $\displaystyle \frac{3}{5}$

  3. $\displaystyle \frac{19}{27}$

  4. $\displaystyle \frac{8}{5}$


Correct Option: A
Explanation:

We know, $S _{\infty }=\dfrac{a}{1-r}$
From the given series, $a=\dfrac{2}{3} ,r=-\dfrac{2}{3}$
$\therefore S _{\infty }=\dfrac{\frac{2}{3}}{1-\left ( -\frac{2}{3} \right )}$


$\Rightarrow \dfrac{\frac{2}{3}}{1+\frac{2}{3}}$

$\Rightarrow \dfrac{\frac{2}{3}}{\frac{5}{3}}$

$\Rightarrow \dfrac{2}{5}$

Sum the series: $\displaystyle {1\, -\, \frac{1}{3}\, +\, \frac{1}{3^2}\, -\, \frac{1}{3^3}\, +\, \frac{1}{3^4}.......\infty}$

  1. $\displaystyle \frac{3}{4}$

  2. $\displaystyle \frac{4}{3}$

  3. $\displaystyle \frac{2}{3}$

  4. $\displaystyle \frac{1}{3}$


Correct Option: A
Explanation:

We know, $S _{\infty }=\dfrac{a}{1-r}$
From the given series, $a=1 ,r=-\dfrac{1}{3}$
$\therefore S _{\infty }=\dfrac{1}{1-\left ( -\dfrac{1}{3} \right )}$
$= \dfrac{1}{1+\dfrac{1}{3}}$


$ =\dfrac{1}{\dfrac{4}{3}}$

$= \dfrac{3}{4}$

If a, b and c are in geometric progression, then $a^2$, $b^2$ and $c^2$ are in _____ progression.

  1. AP

  2. GP

  3. HP

  4. AGP


Correct Option: B
Explanation:

Given $ a,b,c $ are in GP.
So, the common ratio between the first and second term ; second and third will be the same.
$ => \dfrac {b}{a} = \dfrac {c}{b} $


$ => b^2 = ac $

If we square both sides, we see that
$ (b^2)^2 = a^2 \times c^2 $

This means, even, $ a^2, b^2, c^2 $ are also in GP.

The sequence $-6 + 42 - 294 + 2058$ is a

  1. finite geometric sequence

  2. finite arithmetic sequence

  3. infinite geometric sequence

  4. infinite harmonic sequence


Correct Option: A
Explanation:

The sequence $-6 + 42 - 294 + 2058$ is a finite geometric sequence.
Here the common ratio is $-7$.

As it has finite terms, therefore the series is finite sequence.

The sum of the series $10 - 5 + 2.5 - 1.25.....$ is called

  1. finite geometric sequence

  2. finite arithmetic sequence

  3. infinite geometric sequence

  4. infinite harmonic sequence


Correct Option: C
Explanation:

Given series is $10-5+2.5-1.25.....$

Here the common ratio is $\dfrac {-5}{10}=-\dfrac {1}{2}$.
It is also never ending and continued.
Hence, the given series is infinite geometric series.

When a number $x$ is subtracted from each of the numbers $8, 16$, and $40$, the resulting three numbers form a geometric progression. Find the value of $x$.

  1. $3$

  2. $4$

  3. $6$

  4. $12$

  5. $18$


Correct Option: B
Explanation:

Given that ${(16-x)}^{2}=(8-x)(40-x)$
$\Rightarrow 256-32x+{x}^{2} = 320-48x+{x}^{2}$
$\Rightarrow 16x = 64$ 

$\Rightarrow x = 4$

Say true or false.
The total savings (in $Rs.$) after every month for $10$ months when $Rs. 50$ are saved each month are $50, 150, 200, 250, 300, 350, 400, 450, 500$ represent G.P.

  1. True

  2. False


Correct Option: B
Explanation:

Ratio of second term to first term is $ \frac {150}{50} = 3$

Ratio of third term to second term is  $ \frac {200}{150} = 1.33$

Thus, the ratio is not matching. 

Hence, it is not a GP.

Say true or false.
Given series:
$15, 30, 60, 120, 240$ is in G.P.

  1. True

  2. False


Correct Option: A
Explanation:

Given series is $15,30,60,120,240$
Ratio between first two terms $=$ $\dfrac{30}{15}$ $=2$
Ratio between second and third terms $=$ $\dfrac{60}{30}$ $=2$
Ratio between third and fourth terms $=$ $\dfrac{120}{60}$ $=2$
Ratio between fourth and fifth terms $=$ $\dfrac{240}{120}$ $=2$
Since, the ratio between the terms is the same. The series forms a G.P.

Which of the following is not a G.P.?

  1. $2, 4, 6, 8....$

  2. $5, 25, 125, 625....$

  3. $1.5, 3.0, 6.0, 12.0....$

  4. $8, 16, 24, 32, ....$


Correct Option: A,D
Explanation:

In series $2,4,6,8,....$ difference is same i.e. $2$

In $8,16,24,32,......$ difference again is same $8$
$\therefore$ both the series (a) and (b) are in AP as the difference between their consecutive terms is the same.

For the infinite series $1-\cfrac { 1 }{ 2 } -\cfrac { 1 }{ 4 } +\cfrac { 1 }{ 8 } -\cfrac { 1 }{ 16 } -\cfrac { 1 }{ 32 } +\cfrac { 1 }{ 54 } -\cfrac { 1 }{ 128 } -....\quad $ let $S$ be the (limiting) sum. Then $S$ equals

  1. $0$

  2. $\cfrac { 2 }{ 7 } $

  3. $\cfrac { 6 }{ 7 } $

  4. $\cfrac { 9 }{ 32 } $

  5. $\cfrac { 27 }{ 32 } $


Correct Option: B
Explanation:

Combine the terms in threes, to get the geometric series
$\cfrac { 1 }{ 4 } +\cfrac { 1 }{ 32 } +\cfrac { 1 }{ 256 } +....;\quad \quad S=\cfrac { \cfrac { 1 }{ 4 }  }{ 1-\cfrac { 1 }{ 8 }  } =\cfrac { 2 }{ 7 } $ or
rearrange the terms into three series:
$1+\cfrac { 1 }{ 8 } +\cfrac { 1 }{ 64 } +...\quad -\cfrac { 1 }{ 2 } -\cfrac { 1 }{ 16 } -\cfrac { 1 }{ 128 } -....,\quad -\cfrac { 1 }{ 4 } -\cfrac { 1 }{ 32 } -\cfrac { 1 }{ 256 } -....\quad $
${ S } _{ 1 }=\cfrac { 1 }{ 1-\cfrac { 1 }{ 8 }  } =\cfrac { 8 }{ 7 } ;{ S } _{ 2 }=\cfrac { -\cfrac { 1 }{ 2 }  }{ 1-\cfrac { 1 }{ 8 }  } =-\cfrac { 4 }{ 7 } ;{ S } _{ 3}=\cfrac { -\cfrac { 1 }{ 4 }  }{ 1-\cfrac { 1 }{ 8 }  } =-\cfrac { 2 }{ 7 } ;\quad \therefore S=\cfrac { 2 }{ 7 } $

What is the geometric mean of $6$ and $24$ ?

  1. $10$

  2. $12$

  3. $14$

  4. $16$


Correct Option: B
Explanation:
Assume $a, b, c$ in G.P.
then $\dfrac{b}{a}=\dfrac{c}{b}$

$\Rightarrow b^2=ac$

$\Rightarrow b=\sqrt{ac}$

so, $G.M=\sqrt{ac}$

$=\sqrt{6\times 24}$

$G.M =12$

$a^x=b, b^y=c, c^z=a$
Find the value of x, y, z.

  1. $1$

  2. $Not$  $valid$

  3. $-1$

  4. $0$


Correct Option: A
Explanation:

${ a }^{ x }=b\ { b }^{ y }=c=>{ a }^{ xy }=c\ { c }^{ z }=a\ { c }^{ z }=a=>{ b }^{ yz }=a\ \left( { a }^{ x } \right) ^{ yz }=a\ { a }^{ xyz }=a\ xyz=1$

If there exists a geometric progression containing 27, 8 and 12 as three of its terms (not necessarily consecutive) then no. of progressions possible are

  1. $1$

  2. $2$

  3. infinite

  4. None of these


Correct Option: C
Explanation:
$8, 12, 29$

Let the first term be $8$

$\therefore a = 8$    ...(i)

Let the $p^{th}$ ther be $12$

$\therefore ar^{p-1} = 12$     ...(ii)

and $q^{th}$ term be $27$

$\therefore ar^{q-1} = 27$    ...(iii)

(i) / (ii)

$\dfrac{a}{ar^{p-1}} = \dfrac{8}{12}$

$\Rightarrow r^{p-1} = \dfrac{3}{2} = 1.5$      ...(iv)

(iii) / (i)

$r^{q-1} = \dfrac{27}{8} = \left(\dfrac{3}{2}\right)^3$

$r^{q-1} = (1.5)^3$    ...(v)

(v) / (iv)

$\Rightarrow \dfrac{r^{q-1}}{r^{p-1}} = \dfrac{(1.5)^3}{1.5}$

$\Rightarrow r^{q-1-p+1} = (1.5)^2$

$\Rightarrow r^{q-p} = (1.5)^2$

              $= (r^{p-1})^2$     ...from (iv)

$\Rightarrow r^{q-p} = r^{2p-2}$

$\therefore q-p = 2p-2$

$\Rightarrow \boxed{q=3p-2}$

for every distinct value of '$p$' there will be a district integer value of '$q$'

$\therefore $ Infinite no. of progression are possible.

option $C$
- Hide questions